please help me with geometry question :)

Please Help Me With Geometry Question :)

Answers

Answer 1

Given:

The graph of a triangle ABC.

A line is parallel to AC and passes through the point D.

To find:

The point at which the intersection of this parallel line be with the line BC.

Solution:

From the given figure it is clear that A(1,7), B(9,7), C(9,1) and D(5,7).

Slope of AC is

[tex]Slope=\dfrac{y_2-y_1}{x_2-x_1}[/tex]

[tex]Slope=\dfrac{1-7}{9-1}[/tex]

[tex]Slope=\dfrac{-6}{8}[/tex]

[tex]Slope=\dfrac{-3}{4}[/tex]

Slope of parallel lines are same. So, the slope of parallel line is also [tex]-\dfrac{3}{4}[/tex].

The parallel line passes through D(5,7) with slope [tex]-\dfrac{3}{4}[/tex]. So, the equation of the line is

[tex]y-y_1=m(x-x_1)[/tex]

[tex]y-7=-\dfrac{3}{4}(x-5)[/tex]

[tex]y=-\dfrac{3}{4}(x)+\dfrac{15}{4}+7[/tex]

[tex]y=-\dfrac{3}{4}(x)+\dfrac{28+15}{4}[/tex]

[tex]y=-\dfrac{3}{4}(x)+\dfrac{43}{4}[/tex]              ...(i)

From the given graph it is clear that the line BC is a vertical line. So, the x-coordinates of all the points lie on BC are the same, i.e., 9.

Putting x=9, we get

[tex]y=-\dfrac{3}{4}(9)+\dfrac{43}{4}[/tex]

[tex]y=\dfrac{-27}{4}+\dfrac{43}{4}[/tex]

[tex]y=\dfrac{16}{4}[/tex]

[tex]y=4[/tex]

Therefore, the point at which the intersection of this parallel line be with the line BC is (9,4).


Related Questions

Find the greatest common factor of 21x^4 and 49x^3

Answers

The greatest common factor of two terms given as 21x⁴ and 49x³ is equal to  7x³.

To find the greatest common factor (GCF) of two terms, we need to find the highest factor that is common to both terms. In this case, we have 21x⁴ and 49x³.

To find the factors, we can break each term down into its prime factors:

21x⁴ = 3 * 7 * x * x * x * x

49x³ = 7 * 7 * x * x * x

The common factors are 7 and x³. To find the GCF, we multiply these common factors together:

GCF = 7  * x³

GCF = 7x³

To learn more about greatest common factor click on,

https://brainly.com/question/9767511

#SPJ1

HELP PLEASE ASAP PHOTO INCLUDED

Answers

The volume of water that the pool can hold is  150.72 cubic centimeters.

How to find the volume of the pool?

Remember that the volume of a cylinder of radius R and height H is:

V = pi*R²*H

Where pi = 3.14

For the pool we know that the height is H = 3ft, and the diameter is 8ft, then the radius is R = 8ft/2 = 4ft

Replacing that in the volume formula we will get:

V = 3.14*(4cm)²*3cm

V = 150.72 cubic centimeters.

Learn more about cylinders at:

https://brainly.com/question/9554871

#SPJ1

Identify the part, percent and base.

$37.60 interest earned on a $460 investment at 8%.

Part:

Percent:

Base:

Answers

The given information is,$37.60 interest earned on a $460 investment at 8%.

Part:The amount of interest earned is called the part. Here, the amount of interest earned is $37.60. Hence the part is $37.60.

Percent:The rate per 100 units of the base is called the percent. Here, the percent rate is 8%.

Base:The original amount invested or borrowed is called the base. Here, the original investment amount is $460. Hence the base is $460.

Thus,

Part: $37.60

Percent: 8%

Base: $460.

Part: The part is the interest earned, which is $37.60.

Percent: The percent is 8%, which represents the annual interest rate.

Base: The base is the amount of the investment, which is $460.

please help me important question in image

Answers

Segment BF is 36 will be the accurate statement.

Similarity theorem of triangle

A triangle known to be similar if the ratio of similar sides of the triangle is equal to a constant.

From the given triangle, the following expression is true:

EC/FC = AC/BF+FC

Substitute the given parameters into the formula to have:

20/30 = 24+20/BF+30

20/30 = 44/BF+30

2/3 = 44/BF + 30

2(BF+30) = 132

2BF + 60 = 132

2BF = 132 - 60

2BF = 72

BF = 36

Hence the correct statement will be that segment BF is 36

Learn more on similar triangle here:

#SPJ1

What the meaning of statement this?

Answers

The statement means that an n-ary operation on set X takes n inputs and produces a single output, with the operation defined by the function f:[tex]X^n[/tex] -> X.

The statement "An n-ary operation on x is functions f:X^n -> X" can be understood as follows:

In mathematics, an n-ary operation refers to an operation that takes n inputs and produces a single output. In this case, the operation is defined on a set X, and the inputs are elements from X. The result of the operation is also an element from X.

The symbol "f" represents the function that performs the n-ary operation. The function f takes n arguments, which are elements from the set X. The notation [tex]X^n[/tex] indicates the Cartesian product of X with itself n times, meaning that the inputs are taken from X and repeated n times as ordered tuples. The arrow "->" signifies that the function f maps the n-tuple of inputs to a single element in X.

To put it simply, the statement is describing a mathematical operation that takes n elements from a set X and combines them in some way to produce a single element, where the combination is determined by the function f. This function f can be defined explicitly based on the specific operation being considered.

for such more question on function

https://brainly.com/question/14723549

#SPJ8

Nancy flew home from vacation with a heavy bag. With the first airline she flew, Nancy had to pay $20 to check her bag, plus $5 for every pound that her bag was over the weight limit. The next flight was with another airline that had the same weight limit. Nancy had to pay $4 per pound that her bag was over the weight limit, in addition to the checked bag fee of $25. By coincidence, the fees ended up being the same with both airlines. How far over the weight limit was the bag? Write a system of equations, graph them, and type the solution.​

Answers

Nancy's bag was 0.2kg over the weight limit.

What weight in which Nancy's bag exceeded limit?

Let's denote the weight limit as x.

The total cost for the first airline can be expressed as:

Cost1 = 20x + 5

The total cost for the second airline is given by:

Cost2 = 25x + 4

As fees were same with both airlines, we will set up equation to find the weight difference:

20x + 5 = 25x + 4

20x-25x = 4-5

-5x = -1

x = -1/-5

x = 0.2kg

Read more about weight

brainly.com/question/28571689

#SPJ1

The distribution for the life of refrigerators is approximately normal with a mean of 14 years and a standard deviation of 2.5 years. What percentage of refrigerators have lives between 11 years and 18 years?

Answers

The percentage of refrigerators that have lives between 11 years and 18 years is approximately 83.01%.

The values of 11 years and 18 years using the given mean and standard deviation, and then find the area under the standard normal curve between those two standardized values.

First, we standardize the value of 11 years:

z1 = (11 - 14) / 2.5 = -1.2

Next, we standardize the value of 18 years:

z2 = (18 - 14) / 2.5 = 1.6

Now we need to find the area under the standard normal curve between these two standardized values.

We can use a standard normal table or calculator to find this area.

Using a standard normal table, we can find the area between z = -1.2 and z = 1.6 by finding the area to the left of z = 1.6 and subtracting the area to the left of z = -1.2:

Area = P(-1.2 < z < 1.6) = P(z < 1.6) - P(z < -1.2)

Looking up these values in the standard normal table, we find:

P(z < 1.6) = 0.9452

P(z < -1.2) = 0.1151

Substituting these values, we get:

Area = 0.9452 - 0.1151 = 0.8301

For similar questions on percentage of refrigerators

https://brainly.com/question/27234494

#SPJ11

100 Points! Geometry question. Photo attached. Please show as much work as possible. Thank you!

Answers

The proof of ΔRUV ≅ ΔSUT is shown below.

Given that:

RSTV is a rectangle and U is the midpoint of VT.

Here, We have to Prove:

⇒ ΔRUV ≅ ΔSUT

Now, We can prove as;

Statement                               Reason

1) RSTV is a rectangle               Given

2) Angle V and T are                Because every angle in a rectangle is 90

right triangle.

3) ∠V ≅ ∠T                               Because both are right angle.

4) U is the midpoint of VT.        Given

5) VU = UT                                By midpoint theorem.

6) VR ≅ TS                                Opposite sides of rectangle.

7) ΔRUV ≅ ΔSUT                       By SAS congruency theorem.

Learn more about the triangle visit;

brainly.com/question/1058720

#SPJ1

make a table for each equation using the following numbers as : x -2,-1,0,1
y=3x-2

Answers

The equation y = 3x - 2. can be placed in table as shown below. For example, when x = -2, y = 3(-2) - 2 = -8. Similarly, when x = -1, y = 3(-1) - 2 = -5, and so on.

In this table, each row represents a pair of values (x, y) that satisfy the equation y = 3x - 2. For example, when x is -2, y is calculated as 3(-2) - 2 = -8. Similarly, for the other values of x, the corresponding values of y are calculated accordingly.

Here's a table showing the values of x and the corresponding values of y for the equation y = 3x - 2:

x y

-2 -8

-1 -5

0 -2

1 1

To fill in the table, substitute each value of x into the equation and calculate the corresponding value of y. For example, when x = -2, y = 3(-2) - 2 = -8. Similarly, when x = -1, y = 3(-1) - 2 = -5, and so on.

For more such questions on equation , Visit:

https://brainly.com/question/29174899

#SPJ11

Please urgent help I can’t figure this out thank you guys for always helping

Answers

Answer:

Domain:  {-24, -12, -8, 9, 14, 50}

Range:  {23, 69, 83, 92, 97, 32}

Step-by-step explanation:

The domain of a function is the set of all possible input values (usually x) for which the function is defined. The range of a function is the set of all possible output values (usually y, which is synonymous with f(x)) that the function can produce.

For a function like f(x), each coordinate is in the form (x, f(x)) aka (x, y)

Since our x-coordinates are -24, -12, -8, 9, 14, and 50, the domain of f(x) is {-24, -12, -8, 9, 14, 50}

Since our f(x) or y-coordinates are 23, 69, 83, 92, 97, and 32, the range is {23, 69, 83, 92, 97, 32}

Find the quotient and express the answer in scientific notation. 302 ÷ (9.1 x 10^4 )

Answers

The  quotient of 302 ÷ (9.1 x [tex]10^4)[/tex] in scientific notation is approximately 3.31868131868 x [tex]10^1[/tex]

How to find the quotient

Dividing 302 by 9.1 gives:

302 ÷ 9.1 ≈ 33.1868131868

Now, to express this result in scientific notation, we need to move the decimal point to the appropriate position to create a number between 1 and 10. In this case, we move the decimal point two places to the left:

33.1868131868 ≈ 3.31868131868 x[tex]10^1[/tex]

Therefore, the quotient of 302 ÷ (9.1 x [tex]10^4[/tex]) in scientific notation is approximately 3.31868131868 x[tex]10^1[/tex]

Learn more about quotient at https://brainly.com/question/11995925

#SPJ1

increase a gross by 10%​

Answers

The results of increasing a gross by 10% is 158.4.

What is percentage?

Percentage refers to the amount, number or rate of something, regarded as part of a total of 100.

In mathematics, gross is written in figures as 144

increase a gross by 10%

= 144 + (10% × 144)

= 144 + (0.1 × 144)

= 144 + 14.4

= 158.4

Hence, 158.4 is the 10% increase in gross.

Complete question:

Calculate the increase in a gross by 10%.

Read more on percentage:

https://brainly.com/question/843074

#SPJ1

A school class went on a field trip to see a magician perform there were 17 females 20 males in the class the magicians randomly selected a volunteer from the audience in which had 52 females and 68 males given that the randomly selected audience member is a student from the class which equation can be used to find the probability p that the perdimos also a female?please help does anyone know the answer

2
SEE ANSWERS


ASK AN EXPERT

Answers

The probability p that the perdimos is also a female is 17/120

What is Conditional Probability?

Conditional Probability is the possibility or likelihood of an event or outcome happening, based on the existence of a previous event or outcome.

How to determine this

When there were 17 females in class

And 20 male in class

When randomly selected, the female are 52

And males are 68

To calculate the selected volunteer is a student of the class.

= 20 + 17/52 + 68

= 37/120

To calculate the probability that the selected student is female.

= 17/52 + 68

= 17/120

= 0.14

Read more about Probability

https://brainly.com/question/31695049

#SPJ1

Need help, show work please

Answers

Using the first two points we can get an exponential model:

[tex]y = (225/23)*(23/15)^x[/tex]

How to find the exponential equation?

The general exponential equation can be written as:

[tex]y = A*(b)^x[/tex]

We can replace any two values of the table to get the system of eqautions:

[tex]15 = A*(b)\\\\23 = A*(b)^2[/tex]

Taking the quotient between the second and the first equation we get:

[tex]23/15 = (A*b^2)/(A*b)\\\\23/15 = b[/tex]

Replacing that on the first equation we will get:

[tex]15 =A*(23/15)\\15*(15/23) = A\\225/23 = A[/tex]

Then an exponential model is:

[tex]y = (225/23)*(23/15)^x[/tex]

LEarn more about exponential equations at:

https://brainly.com/question/11464095

#SPJ1

Select the correct answer from the drop-down menu. The missing term in the denominator is

Answers

Answer: where is the answer choices

Step-by-step explanation:

At the beginning of the week, James has 8 quarts of milk. At the end of the week there are 412 cups of milk left.

How many cups of milk did James drink during the week?

Answers

James drank [tex]$27 \frac{1}{2}$[/tex] cups of milk during the week

To determine how many cups of milk James drank during the week, we need to find the difference between the initial amount of milk and the amount of milk left at the end of the week. The initial amount of milk: 8 quarts = [tex]$8 \times 4 = 32$[/tex] cupsThe remaining amount of milk: [tex]$4 \frac{1}{2}$[/tex] cupsTo find the amount of milk James drank, we subtract the remaining amount from the initial amount: [tex]$32 \text{ cups} - 4 \frac{1}{2} \text{ cups} = 27 \frac{1}{2}$[/tex] cupsThe concept used in this problem is subtraction. By subtracting the amount of milk left at the end of the week from the initial amount of milk, we can determine how much milk James consumed during the week. The difference between these two values represents the amount of milk that James drank.

Therefore, James drank [tex]$27 \frac{1}{2}$[/tex] cups of milk during the week.

NOTE: The given question has wrong information. The correct question is:

"At the beginning of the week, James has 8 quarts of milk. At the end of the week, there are [tex]$4 \frac{1}{2}$[/tex] cups of milk left. How many cups of milk did James drink during the week?"

For more questions on cups of milk:

https://brainly.com/question/21469890

#SPJ8

There were 104 females and 78 males present at the high school pep rally. Find the ratio of males to the total number of people present. Express as a simplified ratio.

Answers

Answer:

The ratio is 3:7

Step-by-step explanation:

You add 104+78

The un-simplified ratio is 78:182

hope this helps!!

rate this if it does!

LUUK al uit grapii velow.
Part B
-4
Part A
-3-2
Part C
3
2
-2
-3
Part D
Which part of the graph best represents the solution set to the system of
inequalities y ≥x+1 and y + x>-1? (5 points)

Answers

The solution set of given inequalities are represented by Part A.

The given inequalities are

⇒ y ≥ x + 1 and y + x > -1

Hence, The related equations of both inequalities are

y = x + 1

Put x=0, to find the y-intercept and put y=0, to find x intercept.

y = 0 + 1

y = 1

And, 0 = x + 1

x = - 1

Therefore, x-intercept of the equation is (-1,0) and y-intercept is (0,1).

Similarly, for the second related equation

y + x = - 1

y + 0 = - 1

y = - 1

0 + x = - 1

x = - 1

Therefore x-intercept of the equation is (-1,0) and y-intercept is (0,-1).

Now, join the x and y-intercepts of both lines to draw the line.

Now check the given inequalities by (0,0).

0 ≥ 0 + 1

0 ≥ 1

It is a false statement, therefore the shaded region is in the opposite side of origin.

0 + 0 ≥ - 1

0 ≥ - 1

It is a true statement, therefore the shaded region is about the origin.

Hence, From the below figure we can say that the solution set of given inequalities are represented by Part A.

Learn more about the inequality visit:

https://brainly.com/question/25944814

#SPJ1

A thermometer reading 10°C is brought into a room with a constant temperature of 36°C. if the thermometer reads 14°C after 2 minutes, what will it read after beint left in the room for 4 minutes? and for 9 minutes?​

Answers

Answer:

4 minutes: 17.4 °C9 minutes: 23.7 °C

Step-by-step explanation:

You want to know a thermometer's reading 4 minutes and 9 minutes after begin brought into a room with a temperature of 36 °C if its initial reading is 10 °C, and it rises to 14 °C after 2 minutes.

Newton's law of cooling

Newton's law of cooling tells you the temperature difference of 36 -10 = 26 °C will decline exponentially. If it declines to 36 -14 = 22 °C after 2 minutes, then the temperature reading can be modeled by ...

  T = 36 -26·(22/26)^(t/2)

At times of t=4 and t=9, the temperature readings will be ...

4 minutes: 36 -26(11/13)^(4/2) ≈ 17.4 °C9 minutes: 36 -26(11/13)^(9/2) ≈ 23.7 °C

__

Additional comment

The time constant of this thermometer is about 12 minutes, so it will take about 67 minutes to read within 0.1 °C of the room temperature.

<95141404393>

Help Quickly! Which line in the graph contains the point (4,3) and has a slope of 2/5?

*Giving Brainlyest if correct*

A. line r


B. line p


C. line s


D. line t

Answers

Line R if I’m correct

x + 2y = 2 y = −2x − 2

Answers

The solution of the system of equations is (-2, 2).

How to solve the system of equations?

Here we want to find the solution of the system of equations:

x + 2y = 2

y = -2x- 2

To solve this, we can replace the second equation into the first one, then we will get:

x + 2*(-2x - 2) = 2

Now we can solve that for x:

x - 4x - 4 = 2

-3x = 2 + 4

-3x = 6

x = 6/-3 = -2

Thus, the value of y is:

y = -2*-2 - 2

y = 4 - 2 = 2

The solution of the system is (-2, 2)

Learn more about systems of equations at:

https://brainly.com/question/13729904

#SPJ1

Complete question:

"Which is the solution of the system of equations?

x + 2y = 2

y = -2x- 2"

Suppose you pick 4 cards randomly from a well shuffled standard deck of 52 playing cards. The probability that you draw the 2,4,6, and 8 of spades in that order is

Answers

The probability of drawing the 2, 4, 6, and 8 of spades in that specific order is 0.0002.

What is the probability?

The probability of drawing a specific card without replacement from a well-shuffled deck is 1/52 for the first card, 1/51 for the second card, 1/50 for the third card, and 1/49 for the fourth card.

We wish to draw the specific cards (2, 4, 6, and 8 of spades) in that specific order.

The probability will be:

Probability = (1/52) * (1/51) * (1/50) * (1/49)

probability = 0.0002.

Learn more about probability at: https://brainly.com/question/13604758

#SPJ1

You are holding a kite string in your hand and your hand is 3 feet above the ground. The kite is 225 feet above the ground and the angle of elevation from your hand to the kite is 42°. How long is the kite string?

Answers

The kite string is 331.77 feet long.

How to find how long the kite string is?

Trigonometry deals with the relationship between the ratios of the sides of a right-angled triangle with its angles.

Check the attached image for labeling. Where l represents the length of kite string.

h = 225 - 3 = 222 ft

Using trig. ratio:

sin 42° = 222/l

l = 222 / sin 42°

l = 331.77 feet

Learn more about Trigonometry on:

brainly.com/question/11967894

#SPJ1

The side of a square is measured to be 16 ft with a possible error of ±0.1 ft. Use linear approximation or differentials to estimate the error in the calculated area. Include units in your answer.

Answers

The estimated error in the calculated area is approximately 3.2ft²

How did we estimate the error?

To estimate the error in the calculated area of a square, use linear approximation or differentials.

The area of a square is given by the formula A = s², where s represents the length of a side.

Calculate the area of the square using the given side length of 16 ft:

A = (16 ft)²

A = 256 ft²

Now, let's consider the possible error in the side length, which is ±0.1 ft. Treat this error as a change in the side length (Δs) and use linear approximation or differentials to estimate the corresponding change in the area (ΔA).

Using linear approximation, express the change in the area as:

ΔA ≈ 2s Δs

Substituting the values:

ΔA ≈ 2(16 ft)(0.1 ft)

ΔA ≈ 3.2 ft²

Therefore, the estimated error in the calculated area is approximately 3.2ft².

learn more about area of a square: https://brainly.com/question/11444061

#SPJ1

Does the data follow a normal distribution? Why or why not?

Answers

Yes. It follows the normal bell curve

Question 14 of 15
Which number line shows the solutions to x+8 > 15?

Answers

A number line that shows the solutions to x+8 > 15 include the following: B. number line B.

What is an inequality?

In Mathematics and Geometry, an inequality simply refers to a mathematical relation that is typically used for comparing two (2) or more numerical data and variables in an algebraic equation based on any of the inequality symbols;

Greater than (>).Less than (<).Greater than or equal to (≥).Less than or equal to (≤).

Based on the information provided above, we have the following equation (inequality);

x + 8 > 15

By subtracting 8 from both sides of the equation (inequality), we have;

x + 8 - 8 > 15 - 8

x > 7

This ultimately implies that, the inequality must be shaded to the right with an open dot because the inequality symbol is greater than (>).

Read more on inequality here: https://brainly.com/question/30665021

#SPJ1

Missing information:

The question is incomplete and the complete question is shown in the attached picture.

the box plot below represents some data set. what percentage of the data values are greater than 130​

Answers

The percentage of the data values that are greater than 130 is 25%

What percentage of data values are greater than 130

From the question, we have the following parameters that can be used in our computation:

The box plot

From the box plot, the five-number summary are

40, 70, 110, 130, and 150

In the above five-number summary, we have

Third quartile = 130

This means that the percentage of the data values that are greater than 130 is 25%

Read more about box plot at

https://brainly.com/question/3473797

#SPJ1

find the surface area of a regular hexagonal pyramid with the base edges of 10 and a slant height of 9 inches.

Answers

The surface area of the regular hexagonal pyramid is equal to 529.8 in² to the nearest tenth.

How to calculate for the surface area of the regular hexagonal pyramid

Area of regular polygon = 1/2 × apothem × perimeter

perimeter = (s)side length of nonagon × (n)number of side

apothem = s/[2tan(180/n)]

apothem = 10in[2tan(180/6)]

apothem = 5√3in

perimeter = 6 sides × 10in = 60 in²

Area of the bottom hexagon = 1/2 × 5√3 × 60 in²

Area of the bottom hexagon = 259.8075 in²

Area of one triangle side face = 1/2 × 10in × 9in = 45 in²

Area of 6 triangle side faces = 6 × 45 in² = 270 in²

Surface area of the regular pyramid = 259.8075 in² + 270 in²

Surface area of the regular pyramid = 529.8 in²

Therefore, the surface area of the regular hexagonal pyramid is equal to 529.8 in²

Read more about area here:https://brainly.com/question/27440983

#SPJ1

Extract the common factor from
(1) 2.x-2.y
(2) 5.x.x.x+5.3.x.y​

Answers

Answer:

(1) 2x - 2y = 2(x - y)

(2) 5x^3 + 15xy = 5x(x^2 - 3y)

Find your answer on above picture ihope it will help you thank you.

Find the 5th term in the expansion of (3x + 1)7 in simplest form.

Answers

The 5th term in the expansion of [tex](3x + 1)^7[/tex] would be [tex]945x^3[/tex].

Binomial expansion

To find the 5th term in the expansion of  [tex](3x + 1)^7[/tex], we use the binomial theorem formula:

[tex](a + b)^n = nC0 a^n b^0 + nC1 a^(n-1) b^1 + nC2 a^(n-2) b^2 + ... + nCn-1 a^1 b^(n-1) + nCn a^0 b^n[/tex]

where nCk represents the binomial coefficient, given by the formula:

nCk = n! / (k!(n-k)!)

In this case, a = 3x and b = 1, so we have:

[tex](3x + 1)^7 = 7C0 (3x)^7 1^0 + 7C1 (3x)^6 1^1 + 7C2 (3x)^5 1^2 + 7C3 (3x)^4 1^3 + 7C4 (3x)^3 1^4 + 7C5 (3x)^2 1^5 + 7C6 (3x)^1 1^6 + 7C7 (3x)^0 1^7[/tex]

To find the 5th term, we need to look at the term with k = 4, which is:

[tex]7C4 (3x)^3 1^4[/tex] = [tex]35 (3x)^3[/tex]

     = [tex]35 (3x)^3[/tex]

     = [tex]35 (27x^3)[/tex]

Therefore, the 5th term in the expansion of [tex](3x + 1)^7[/tex] is [tex]945x^3[/tex].

More on Binomial expansions can be found here: https://brainly.com/question/29260188

#SPJ1

Other Questions
hedging one commodity by using a futures contract on another commodity is called group of answer choices surrogate hedging. correlative hedging. alternative hedging. cross hedging. proxy hedging. Tried to post this last night and no one helped. Please help now. :D The Iliad is a poem about war, the campaign of a large coalition to "punish" an overseas power for infractions against certain norms and standards. Discuss this point with reference to modern history and, especially, events of your own lifetime A small particle has charge -5.00 uC and mass 2.00 x 10^-4 kg. It moves from point A where the electric potential is VA= +200.0 Volts, to point B, where the electric potential is VB= +800.0 Volts. The electric force is the only force acting on the particle. The particle has a speed of 5.00 m/s at point A.What is the speed at Point B? A correlation is computed for a sample of n = 18 pairs of X and Y values. What correlations are statistically significant with ? = .05, two tails. 1. correlations greater than or equal to 0.456 and correlation less than or equal to_____ Identifying the Parts of SpeechDirections: Read the following sentences. Highlight the nouns in blue, pronouns in light blue, adjectives in yellow, verbs in green, adverbs in orange, prepositions in purple, and conjunctions in pink. 1. The swift train drove quickly through the night but still didnt arrive on time. 2. The ugly duckling wished he was a beautiful swan, and his wish came true. 3. The knight in shining armor asked the princess, Rapunzel, to let down her golden hair. 4. The talkative students ate lunch in the park when they should have been in class. 5. The eight-foot alligator took a nap in the sun while I silently paddled by on my kayak. Please just say what color it is thank you all so much. :) in the stomach, ___ chemically break down swallowed food. separate the redox reaction into its component halfreactions. o2 4li2li2o use the symbol e for an electron. according to hans roslings video, what are the two countries that produce the most carbon dioxide? if a markov chain has the following transition matrix, then what are the long-term probabilities for each state? enter exact answers. Question 7 of 20What effect did the mandate system have on national borders after WorldWar I? An ideal gas is compressed isobarically to one-third of its initial volume. The resulting pressure will be?A) three times as large as the initial value. B) equal to the initial value. C) more than three times as large as the initial value. D) nine times the initial value. E) impossible to predict on the basis of this data. Imagine your firm has the short run total cost function: C = q^(3) 3q^(2) + 10q + 250. At what level of output (quantity of production) is your average variable cost (AVC) minimized? variable costing treats blank______ manufacturing costs as product costs. A firm in a competitive market receives $1,000 in total revenue and has marginal revenue of $20.The firms average revenue is $_____??, and _____? units were sold. 4 Starting from rest on a level road a girl can reach a speed of 5 m/s in 10s on her bicycle. a Find the acceleration b Find the average speed during the 10s. c Find the distance she travels in 10 s. [2] Eventually, even though she still pedals as fast as she can, she stops accelerating and her speed reaches a maximum value. Explain in terms of the forces acting why this happens. find (a) the amplitude and (b) the phase constant in the sum y of the following quantities: y1 = 11 sin t y2 = 16 sin(t 33) y3 = 5.0 sin(t - 35) using the phasor method. what is the slope of the line tangent to the curve x3 y3=2x2y2 at the point (1,1 Thirty-six of the staff of 80 teachers at a local intermediate school are certified in cardio-pulmonary resuscitation (cpr). in 180 days of school, what is the mean, variance, and standard deviation of the number of days can we expect that the teacher on bus duty will likely be certified in cpr? A natural food company produces and sells organic almond milk for $9.00 per gallon. This companyestimates its fixed costs to be 1121 dollars per month and the variable cost to produce each gallon ofalmond milk is $7.50.What is the margin of profit if 1494 gallons of almond milk are produced and sold each month?A Select an answer of $